K
Khách

Hãy nhập câu hỏi của bạn vào đây, nếu là tài khoản VIP, bạn sẽ được ưu tiên trả lời.

29 tháng 7 2016

\(\frac{17}{15}=\frac{493}{435};\frac{31}{29}=\frac{465}{435};493>465=>\frac{17}{15}>\frac{31}{29}\)

29 tháng 7 2016

\(\frac{17}{15}\)>\(\frac{31}{29}\)

29 tháng 7 2016

a/\(\frac{1212}{1313}=\frac{12x101}{13x101}=\frac{12}{13}\)

Vậy \(\frac{12}{13}=\frac{1212}{1313}\)

b/Ta có:

\(\frac{4}{5}< 1\)

\(\frac{10}{9}>1\)

=>\(\frac{4}{5}< 1< \frac{10}{9}\Rightarrow\frac{4}{5}< \frac{10}{9}\)

c/\(\frac{17}{15}=1+\frac{2}{15}\)

\(\frac{31}{29}=1+\frac{2}{29}\)

Vì \(\frac{2}{15}>\frac{2}{29}\) nên \(\frac{17}{15}>\frac{31}{29}\)

-------------

chú ý: dấu "=>" có nghĩa là "suy ra"

29 tháng 7 2016

D(=)

E(<)

F(>)

14 tháng 9 2019

(1-3/2014)< (1-3/2017) => 2011/2014<2014/2017 13/27<13/26=1/2 27/53>27/54=1/2 Vì 13/27<1/2<27/53 nên 13/27<27/53 C)phần bù đơn vị 11/15<13/17

26 tháng 2 2017

(1-3/2014)< (1-3/2017)

=> 2011/2014<2014/2017 

13/27<13/26=1/2
27/53>27/54=1/2
Vì 13/27<1/2<27/53 nên 13/27<27/53

c)phần bù đơn vị 11/15<13/17

13 tháng 7 2023

a) \(\dfrac{25}{37}\times\dfrac{18}{29}+\dfrac{18}{29}\times\dfrac{12}{37}\)

\(=\dfrac{18}{29}\times\left(\dfrac{25}{37}+\dfrac{12}{37}\right)\)

\(=\dfrac{18}{29}\times\dfrac{37}{37}\)

\(=\dfrac{18}{29}\times1\)

\(=\dfrac{18}{29}\)

b) \(\dfrac{31}{85}\times\dfrac{11}{19}+\dfrac{31}{85}\times\dfrac{12}{19}-\dfrac{42}{19}\times\dfrac{31}{85}\)

\(=\dfrac{31}{85}\times\left(\dfrac{11}{19}+\dfrac{12}{19}-\dfrac{42}{19}\right)\)

\(=\dfrac{31}{85}\times\dfrac{-19}{19}\)

\(=\dfrac{31}{85}\times-1\)

\(=-\dfrac{31}{85}\)

c) \(\dfrac{16}{53}:\dfrac{17}{9}-\dfrac{16}{53}:\dfrac{17}{8}\)

\(=\dfrac{16}{53}:\left(\dfrac{9}{17}-\dfrac{8}{17}\right)\)

\(=\dfrac{16}{53}:\dfrac{1}{17}\)

\(=\dfrac{16}{901}\)

c) \(\dfrac{1}{5}\times\dfrac{12}{31}\times\dfrac{4}{3}+\dfrac{19}{31}\times\dfrac{4}{15}\)

\(=\dfrac{4}{15}\times\dfrac{12}{31}+\dfrac{19}{31}\times\dfrac{4}{15}\)

\(=\dfrac{4}{15}\times\left(\dfrac{12}{31}+\dfrac{19}{31}\right)\)

\(=\dfrac{4}{15}\times\dfrac{31}{31}\)

\(=\dfrac{4}{15}\times1\)

\(=\dfrac{4}{15}\)

a: =18/29*(25/37+12/37)

=18/29

b: =31/85(11/19+12/19-42/19)

=-31/85

c; =16/53(9/17+8/17)=16/53

d: =4/15(12/31+19/31)=4/15

24 tháng 5 2016

2000/2001<1

2001/2002<1

2002/2003<1

...

2015/2016<1

=>2000/2001+2001/2002+2002/2003+2003/2004+...+2015/2016<1+1+1+1+1+...+1=15

Vậy...   

24 tháng 5 2016

Ta có:

2000/ 2001 < 1

2001/2002 < 1

..................

2015/ 2016<1

=> 200/2001 + 2001/202+...+ 2015/2016 < 1 + 1+1 +1+...+1( 15 số hạng)

=> 200/2001 + 2001/202+...+ 2015/2016< 1 x 15 = 15

29 tháng 7 2016

a.11/16 < 5/4

b.13/14 > 13/15

c.12/13 < 22/33

29 tháng 7 2016

a. vì 11/16<1 ; 5/4 > 1

=> 11/16 < 5/4

b. vì 14<15

=> 13/14 > 13/15

c. có : 22/33=2/3 =26/39; 12/13=36/39

=> 12/13>22/33